Answer (B) is correct . The size of Mulva’s capital budget will be determined by the number of projects it can profitably undertake, i.e., those projects for which the IRR is greater than the applicable weighted average cost of capital. ? First, the cost of each type of capital must be determined. The formula for calculating the cost of retained earnings is k s = (D 1 ÷ P 0) + g, where D 1 equals the dividend after Year 1, P 0 equals the current stock price, and g equals the expected growth rate. The cost of retained earnings is 11.25% [($2.50 ÷ $40) + 0.05]. The formula for calculating the cost of new equity is k e = [(D 1 ÷ (P 0 – floating cost)]+ g. The cost of new equity is 11.58% {[$2.50 ÷ ($40 – $2)] + 0.05}. Given the firm’s target capital structure and its retained earnings balance of $900,000, the firm can raise $1,500,000 with debt and retained earnings before it must use outside equity. Therefore, the WACC for $0 – $1,500,000 of financing is equal to 9.15% [0.4 (0.10)(1 – 0.4) + 0.6(0.1125) = 0.0915]. Above $1,500,000, the firm must issue some new equity, so the WACC is 9.35% [0.4(0.10)(1 – 0.4) + 0.6(0.1158) = 0.0935]. Projects A, B, and C will definitely be undertaken because the IRR is greater than the WACC. Next, determine whether Project D will be profitable. Financing Projects A, B, and C requires $1,200,000 in capital. Therefore, the $550,000 needed for Project D would involve financing $300,000 with debt and retained earnings and $250,000 with debt and equity. Thus, the WACC for Project D is 9.24% {[($300,000 ÷ $550,000) × 0.0915] +25/55*0.0935}
Answer (A) is incorrect because The amount of $1,200,000 does not include a project or projects, which would be profitable since its or their IRR is greater than the weighted average cost of capital.
Answer (C) is incorrect because The amount of $2,400,000 includes a project or projects, which would not be profitable since its or their IRR is less than the weighted average cost of capital.
Answer (D) is incorrect because The amount of $800,000 does not include a project or projects, which would be profitable since its or their IRR is greater than the weighted average cost of capital.
|